0 Daumen
1,9k Aufrufe

unser Tutor ist leider nicht in der Lage uns diese Aufgabe zu erklären und da er uns keine Lösungsansätze geben konnte, kommen wir nicht weiter.

Bild Mathematik

Avatar von

........gelöscht...................

Soll \(y(x)=1-\frac13x^3\) eine Lösung des AWP sein?

Bild Mathematikin der Übung haben wir diesen Ansatz bekommen, leider sehe ich keinen Zusammenhang zwischen Deiner Lösung und diesem Anschrieb.

Was steht in der ersten Zeile zwischen "+....+"  ?

Und soll das y=1-1/3 x^3 das Endergebnis dieser Aufgabe sein?

................gelöscht.......................

Soll jetzt \(y(x)=1-x^3-\frac13(x^7-x^4)\) eine Lösung des AWP sein?

@GL: a2=-x macht keinem Sinn. Die an sind konstante Koeffizienten.

...........gelöscht..............

Das hatte ich auch erst vor, aber das obige vom Fragesteller eingefügte Bild entspricht bereits einer Lösung ;)

..............gelöscht..................

Leute,

Ich will doch nur diese Aufgabe verstehen!

Sorry! Meine Lösung gehört zu einer ähnlichen Aufgabe, jedoch ist diese leider sehr zusammengefasst und unser Übungsleiter konnte uns auch nicht viel dazu sagen, weil er es selbst nicht richtig verstanden hat.

Habe in 2 Wochen eine Klausur und bin für jeden Ansatz dankbar.

@GrosserLoewe: Vielen lieben Dank!!! Lass dich nicht von den anderen ärgern. Bisher habe ich deine Ansätze immer besser als in der Übung verstanden.

Du darfst gerne weiterhin hier schreiben. :)

Danke nochmal!

@Gast jc2144: wenn du meinst, dass ich die richtige Lösung habe, dann würde ich gerne wissen, wie die Zwischenschritte für die oben genannte Aufgabe lauten. Bei meiner Lösung wurde kein AWP berücksichtigt!

1 Antwort

0 Daumen
 
Beste Antwort

Hi,
für eine allg. lineare Dgl. zweiter Ordnung der Form
$$ y''(x) + p(x) y'(x) +q(x) y(x) = 0 $$ mit \( y(0) = y_0 \) und \( y'(0) = y'_0 \) gilt folgendes bei einem Potenzreihenansatz der Form $$ y(x) = \sum_{k=0}^\infty a_k x^k $$
$$ (1) \quad a_{n+2} = -\frac{1}{(n+1)(n+2)} \sum_{k=0}^n \left[ p_k (n+1-k) a_{n+1-k} + q_k a_{n-k} \right] $$
wobei gilt \( p(x) = \sum_{k=0}^\infty p_k x^k \) und \( q(x) = \sum_{k=0}^\infty q_k x^k \)

Für die Koeffizienten \( a_0 \) und \( a_1 \) gilt, \( a_0 = y_0 \) und \( a_1 =y'_0 \)

Wenn man jetzt das obige auf Dein Beispiel anwendet, folgt
$$ a_0 = 1 \ , a_1 = 0 $$ $$ p_0 = p_1 = 0 \ , q_0 = 0 \ , q_1 = 2 $$
Die anderen Koeffizienten der Potenzreihen von \( p(x) \text{ und } q(x) \) sind Null.

Einsetzen in (1) ergibt für \( n = 1 \) folgendes
$$ a_3 = -\frac{1}{6} q_1 a_0 = -\frac{1}{3} $$ Das ist der Term, den auch Oswald berechnet hatte.
Durch nachrechnen erkennt man, dass gilt \( a_4=0 \) und \( a_5=0 \)
Für \( a_6 \) bekommt man folgendes
$$ a_6 = -\frac{1}{30} \left( q_1 a_3 + p_2 a_3 \right) = \frac{1}{3 \cdot 6} $$
Allgemein gilt
$$ a_{3i} = \left[-\frac{1}{3i} \right] \left[-\frac{1}{3(i-1)} \right] ... \left[-\frac{1}{3} \right] a_0 = (-1)^i \frac{a_0}{3^i \cdot i!} = \frac{(-1)^i}{3^i \cdot i!} $$

Damit sieht die Lösung so aus:

$$ y(x) = \sum_{k=0}^\infty a_{3k} x^{3k} = \sum_{k=0}^\infty \frac{(-1)^k}{3^k \cdot k!} x^{3k} $$

Streng genommen muss noch beweisen das die Koeffizient \( a_{3i-1} \) und \( a_{3i-2} \) alle Null sind.

Als ergänzende Links kann man folgende empfehlen



NB: Manchmal gibt es hier im Forum Mitglieder die lieber maulen als helfen.

Avatar von 39 k

Ein anderes Problem?

Stell deine Frage

Willkommen bei der Mathelounge! Stell deine Frage einfach und kostenlos

x
Made by a lovely community